Jump to content

If I increases, doesn't B increase too?


DandelionTheory

Recommended Posts

This is a question about variables having to do with force on a current.

I need correction in my understanding, I don't get the difference when a current is in a wire vs not. 

I have drawn out a scenario, the contraption I drew consists of 5 parts. The positive wire (red), it's insolation with exposure at one end (black), the connecting structure (green), the negative pole(blue), and a power source (inside green).

In this scenario, the positive wire's exposed end is 1mm from the closest end of the negative blue wire. 

The electric field applied is over 3kv to exploit thermionic emission, and induce a spark. If the current was to increase during thermionic emission, would the changing magnetic field push all current away from the center of the current loop?Sketch_20200808_205618.thumb.png.111b2dad3c1adef06755b47bc6afcb18.png

Link to comment
Share on other sites

Force on a current is from an external field. You have not identified such a field.  

 

11 hours ago, DandelionTheory said:

This is a question about variables having to do with force on a current.

I need correction in my understanding, I don't get the difference when a current is in a wire vs not. 

I have drawn out a scenario, the contraption I drew consists of 5 parts. The positive wire (red), it's insolation with exposure at one end (black), the connecting structure (green), the negative pole(blue), and a power source (inside green).

In this scenario, the positive wire's exposed end is 1mm from the closest end of the negative blue wire. 

The electric field applied is over 3kv to exploit thermionic emission, and induce a spark. If the current was to increase during thermionic emission, would the changing magnetic field push all current away from the center of the current loop?Sketch_20200808_205618.thumb.png.111b2dad3c1adef06755b47bc6afcb18.png

“connecting structure” and “power source”?

Nothing here is a current in a wire.

Link to comment
Share on other sites

6 hours ago, swansont said:

Force on a current is from an external field. You have not identified such a field.  

 

“connecting structure” and “power source”?

Nothing here is a current in a wire.

I said 

"In this scenario, the positive wire's exposed end is 1mm from the closest end of the negative blue wire. 

The electric field applied is over 3kv to exploit thermionic emission, and induce a spark."

As a physicist, what additional information is required for the scenario to be understood? I do not see external fields on rail guns.

Link to comment
Share on other sites

18 hours ago, DandelionTheory said:

In this scenario, the positive wire's exposed end is 1mm from the closest end of the negative blue wire.

What gives the red wire a positive charge? It doesn't appear to be connect anything. Or is it supposed to be connected to the green (power source?) through the insulator?

18 hours ago, DandelionTheory said:

The electric field applied is over 3kv to exploit thermionic emission, and induce a spark.

Where are you expecting the spark to occur? Between the points you have labelled "+" and "-"?

And why do you want a spark at all? Why not just complete the circuit with a wire?

18 hours ago, DandelionTheory said:

If the current was to increase during thermionic emission, would the changing magnetic field push all current away from the center of the current loop?

I don't think the magnetic field caused by the current will affect the wire carrying that current. But it would affect another, nearby, current-carrying wire.

This is still pretty crude, but is this why you are trying to represent?

Untitled.png.ef3c2d822c7383d3e7736a531f15629d.png

Link to comment
Share on other sites

1 hour ago, DandelionTheory said:

I said 

"In this scenario, the positive wire's exposed end is 1mm from the closest end of the negative blue wire. 

The electric field applied is over 3kv to exploit thermionic emission, and induce a spark."

As a physicist, what additional information is required for the scenario to be understood? I do not see external fields on rail guns.

A lot.

You refer to “the connecting structure (green)” and “a power source (inside green)” which is where the spark should occur. I asked about them, and you haven’t clarified anything

Thermionic emission comes from a high temperature, not a potential difference

 

Quote

I do not see external fields on rail guns.

No? There’s no electric or magnetic field? How do they work?

 

Link to comment
Share on other sites

57 minutes ago, swansont said:

A lot.

You refer to “the connecting structure (green)” and “a power source (inside green)” which is where the spark should occur. I asked about them, and you haven’t clarified anything

Thermionic emission comes from a high temperature, not a potential difference

616910345_Capture_2020-08-09-16-54-47.thumb.png.91c44e2c0ada9da6a60e38e32a395615.png

Quote

No? There’s no electric or magnetic field? How do they work?

The current in each rail acts on the armature and the opposite rail. The circuit is completed by the armature in a rail gun. in the scenario I presented, the thermionic emission, aka electric arc, substitutes the armature. This would complete the circuit right?

1 hour ago, Strange said:

 

This is still pretty crude, but is this why you are trying to represent?

Untitled.png.ef3c2d822c7383d3e7736a531f15629d.png

Arc and wires would move perpendicular to magnetic field. Rail guns work this way, if the current is in flux over time. When the arc reaches a point past the bounds of the box, the wires have moved while the arc can return to the battery. I would assume this can be repeated. The original scenario had the wires bent away from each other because I assumed the arc would take the shortest path and would be forced downward away from the center of the current loop while the current in the wires would be forced away from the center of the current loop at each given slice of time.

Edited by DandelionTheory
Link to comment
Share on other sites

11 hours ago, DandelionTheory said:

Arc and wires would move perpendicular to magnetic field. Rail guns work this way, if the current is in flux over time.

The rails could push each other apart, and the two rails can push on the projectile. But I'm not sure how or if that applies to a single wire.

11 hours ago, DandelionTheory said:

When the arc reaches a point past the bounds of the box

Where is the box? It doesn't appear in your diagram.

11 hours ago, DandelionTheory said:

the wires have moved while the arc can return to the battery

If the wires move apart then you may not be able to support the arc (depending on voltage, available current, humidity and many other factors) and so the circuit would break.

11 hours ago, DandelionTheory said:

The original scenario had the wires bent away from each other because I assumed the arc would take the shortest path and would be forced downward away from the center of the current loop while the current in the wires would be forced away from the center of the current loop at each given slice of time.

The arc will take the shortest path, which is a straight line. This seems inconsistent with your "forced downward". You need to calculate the strength of the magnetic field generated by the wires and see if it is enough to displace the path of the electrons. (I am guessing not, based purely on the size of the coils and magnets in old CRT tubes)

The current in the wires would remain in the wires, it can't be "forced" anywhere. Do you mean the wires would be deflected by the magnetic field generated by the spark? Any such effect would be tiny. But it depends on how the wires fixed, their stiffness, the current involved, etc.

 

I think this is all too vague (and confusing; you keep introducing new things when answering questions) to make any useful conclusions. You need to be more specific about the voltage, current, length of the wire and their mechanical properties, air pressure, temperature, relative humidity, 

Link to comment
Share on other sites

14 hours ago, DandelionTheory said:

616910345_Capture_2020-08-09-16-54-47.thumb.png.91c44e2c0ada9da6a60e38e32a395615.png

That's Schottky emission, which is a combination of thermal effects and a voltage

There's nothing to prevent you from putting a voltage on this, but that basic description of thermionic emission refers to having the thermal energy exceed the work function. IOW, saying it's an arc discharge is fine (that covers both effects), but specifying that it's thermionic emission is misleading. 

https://en.wikipedia.org/wiki/Thermionic_emission

 

14 hours ago, DandelionTheory said:

 

The current in each rail acts on the armature and the opposite rail. The circuit is completed by the armature in a rail gun. in the scenario I presented, the thermionic emission, aka electric arc, substitutes the armature. This would complete the circuit right?

And that gives you a field.

 

14 hours ago, DandelionTheory said:

Arc and wires would move perpendicular to magnetic field.

What magnetic field? You didn't specify one.

 

 

Link to comment
Share on other sites

On 8/10/2020 at 4:24 AM, Strange said:

The rails could push each other apart, and the two rails can push on the projectile. But I'm not sure how or if that applies to a single wire.

In the original scenario, the wires are bent away from each other; up is also a direction the wires are forced.

On 8/10/2020 at 4:24 AM, Strange said:

Where is the box? It doesn't appear in your diagram.

This was in reference to the picture strange posted.

On 8/10/2020 at 4:24 AM, Strange said:

If the wires move apart then you may not be able to support the arc (depending on voltage, available current, humidity and many other factors) and so the circuit would break.

Agreed. The initial spark is a factor of humidity, voltage, atmosphere pressure, dielectric breakdown, and some more. From what I've learned about Jacobs ladder papers, the current supplied maintains the arc.

On 8/10/2020 at 4:24 AM, Strange said:

The arc will take the shortest path, which is a straight line. This seems inconsistent with your "forced downward". You need to calculate the strength of the magnetic field generated by the wires and see if it is enough to displace the path of the electrons.

Perfect. That's the answer. But "forced away from the center of the current loop" is more specific.

If the initial arc is at shortest path, maintain arc with increased current, which increases magnetic field and displaces arc and current in wires. When arc's contact with the ground wire reaches the length of the ground wire, disengage current supplied and calculate total displacement.

I cannot assume further if this isn't agreed upon as the intent of the op.

Edited by DandelionTheory
Link to comment
Share on other sites

4 hours ago, DandelionTheory said:
On 8/10/2020 at 12:24 PM, Strange said:

Where is the box? It doesn't appear in your diagram.

This was in reference to the picture strange posted.

There is no box in my diagram, either! 🙂

4 hours ago, DandelionTheory said:

Agreed. The initial spark is a factor of humidity, voltage, atmosphere pressure, dielectric breakdown, and some more. From what I've learned about Jacobs ladder papers, the current supplied maintains the arc.

But if you increase the gap, you will rapidly stop the spark. Unless you keep increasing the voltage. (Operating at a lower pressure might help.)

4 hours ago, DandelionTheory said:

Perfect. That's the answer. But "forced away from the center of the current loop" is more specific.

If the initial arc is at shortest path, maintain arc with increased current, which increases magnetic field and displaces arc and current in wires. When arc's contact with the ground wire reaches the length of the ground wire, disengage current supplied and calculate total displacement.

If you force the electrons to take a longer path in the arc, then you will need even higher voltage to maintain the arc.

This does not necessarily mean that the current will increase. Calculating the resistivity of a plasma is extremely complicated. I suspect (but I really don't know) that the current would be roughly constant if you increase both the voltage and the spark gap.

4 hours ago, DandelionTheory said:

In the original scenario, the wires are bent away from each other

If they are at 90º (as it seems from your diagram) then the force between them will be zero.

(And, generalising from that, I *think* the net force within a closed loop will be zero -- but my math skills aren't up to proving that, or I don't have time, at least!)

Link to comment
Share on other sites

16 hours ago, Strange said:

There is no box in my diagram, either! 

Oh, haha. 

16 hours ago, Strange said:

If you force the electrons to take a longer path in the arc, then you will need even higher voltage to maintain the arc.

That too.

16 hours ago, Strange said:

This does not necessarily mean that the current will increase. Calculating the resistivity of a plasma is extremely complicated. I suspect (but I really don't know) that the current would be roughly constant if you increase both the voltage and the spark gap.

I'm not calculating current increase, the question is: does the magnetic field increase due to current increase maintaining the spark gap, and does that additional current increase in the spark gap translate to force on the wires?

I'm heavily leaning on current due to this 

1545751676_Capture_2020-08-10-06-27-23.thumb.png.8b9cf3b5e2f9f5ae65920d7fae70453a.png

Edited by DandelionTheory
Link to comment
Share on other sites

8 hours ago, DandelionTheory said:

I'm not calculating current increase, the question is: does the magnetic field increase due to current increase maintaining the spark gap, and does that additional current increase in the spark gap translate to force on the wires?

You need to increase the voltage to maintain the arc. That will not necessarily increase the current.

You may need to increase the voltage further to increase the current. 

How the current relates to voltage and size of gap is beyond me.

And what force this might generate is beyond me too.

8 hours ago, DandelionTheory said:

I'm heavily leaning on current due to this 

Can you provide the source?

Link to comment
Share on other sites

2 hours ago, Strange said:

Can you provide the source?

I found it through Google search:

https://www.google.com/url?sa=t&source=web&rct=j&url=https://web5.uottawa.ca/www10/every_2012/document/20120404083003001.pdf&ved=2ahUKEwi7muHPhpjrAhXkLX0KHY5QCyoQFjAfegQIBxAB&usg=AOvVaw3_RHdoHQ4dwiVhXQq_5FB9

2 hours ago, Strange said:

You need to increase the voltage to maintain the arc. That will not necessarily increase the current.

You may need to increase the voltage further to increase the current.

Thank you.

2 hours ago, Strange said:

And what force this might generate is beyond me too

Do you know what direction I would need to go to find that out? 

Link to comment
Share on other sites

Create an account or sign in to comment

You need to be a member in order to leave a comment

Create an account

Sign up for a new account in our community. It's easy!

Register a new account

Sign in

Already have an account? Sign in here.

Sign In Now
×
×
  • Create New...

Important Information

We have placed cookies on your device to help make this website better. You can adjust your cookie settings, otherwise we'll assume you're okay to continue.